Integral Method to find upper and lower bounds on the sum












0












$begingroup$


I got this problem wrong on my exam and I know why I got it wrong but I am not sure how to find the right answer.



This is the question:
Consider$sum_{k=1}^n (3+ 2/k^3)$. Use the integral method to obtain upper and lower bound on the sum. Do not split the summation into $sum_{k=1}^n 3+sum_{k=1}^n ( 2/k^3)$



This is the formula we had to use: Click here to see the approximation integral formula



This function is decreasing and I was able to calculate the lower bound, but I am not sure how to do the upper bound becuase this is what I get $ int_{0}^{n}$(3+2/k^3)dx and after taking the antiderivative. I get 3k - (1/k^2) from 0 to n. so if I take the integral from 0 to n then 1/k^2 becomes undefined. I tried to change the integral from -1 to n but I got it wrong on the exam.



Could someone please tell me how to solve this? Thanks










share|cite|improve this question









$endgroup$

















    0












    $begingroup$


    I got this problem wrong on my exam and I know why I got it wrong but I am not sure how to find the right answer.



    This is the question:
    Consider$sum_{k=1}^n (3+ 2/k^3)$. Use the integral method to obtain upper and lower bound on the sum. Do not split the summation into $sum_{k=1}^n 3+sum_{k=1}^n ( 2/k^3)$



    This is the formula we had to use: Click here to see the approximation integral formula



    This function is decreasing and I was able to calculate the lower bound, but I am not sure how to do the upper bound becuase this is what I get $ int_{0}^{n}$(3+2/k^3)dx and after taking the antiderivative. I get 3k - (1/k^2) from 0 to n. so if I take the integral from 0 to n then 1/k^2 becomes undefined. I tried to change the integral from -1 to n but I got it wrong on the exam.



    Could someone please tell me how to solve this? Thanks










    share|cite|improve this question









    $endgroup$















      0












      0








      0





      $begingroup$


      I got this problem wrong on my exam and I know why I got it wrong but I am not sure how to find the right answer.



      This is the question:
      Consider$sum_{k=1}^n (3+ 2/k^3)$. Use the integral method to obtain upper and lower bound on the sum. Do not split the summation into $sum_{k=1}^n 3+sum_{k=1}^n ( 2/k^3)$



      This is the formula we had to use: Click here to see the approximation integral formula



      This function is decreasing and I was able to calculate the lower bound, but I am not sure how to do the upper bound becuase this is what I get $ int_{0}^{n}$(3+2/k^3)dx and after taking the antiderivative. I get 3k - (1/k^2) from 0 to n. so if I take the integral from 0 to n then 1/k^2 becomes undefined. I tried to change the integral from -1 to n but I got it wrong on the exam.



      Could someone please tell me how to solve this? Thanks










      share|cite|improve this question









      $endgroup$




      I got this problem wrong on my exam and I know why I got it wrong but I am not sure how to find the right answer.



      This is the question:
      Consider$sum_{k=1}^n (3+ 2/k^3)$. Use the integral method to obtain upper and lower bound on the sum. Do not split the summation into $sum_{k=1}^n 3+sum_{k=1}^n ( 2/k^3)$



      This is the formula we had to use: Click here to see the approximation integral formula



      This function is decreasing and I was able to calculate the lower bound, but I am not sure how to do the upper bound becuase this is what I get $ int_{0}^{n}$(3+2/k^3)dx and after taking the antiderivative. I get 3k - (1/k^2) from 0 to n. so if I take the integral from 0 to n then 1/k^2 becomes undefined. I tried to change the integral from -1 to n but I got it wrong on the exam.



      Could someone please tell me how to solve this? Thanks







      definite-integrals approximate-integration






      share|cite|improve this question













      share|cite|improve this question











      share|cite|improve this question




      share|cite|improve this question










      asked Apr 2 '17 at 9:35









      jenkins342jenkins342

      4




      4






















          1 Answer
          1






          active

          oldest

          votes


















          0












          $begingroup$

          Hint. You may write
          $$
          begin{align}
          sum_{k=1}^nleft(3+frac1{k^3} right)&=left(3+frac1{1^3} right)+sum_{k=color{red}{2}}^nleft(3+frac1{k^3} right)
          \\&le left(3+frac1{1^3}right)+int_{color{red}{2-1}}^nleft(3+frac1{x^3} right)dx
          end{align}
          $$ then getting easily an upper bound.






          share|cite|improve this answer









          $endgroup$













          • $begingroup$
            thanks! Will I need to calculate the derivative of 5 in the integration part or just take the derivative of what's inside the summation and then multiply by 5? Also, for the lower bound, I can use the original summation without changing the bounds?
            $endgroup$
            – jenkins342
            Apr 2 '17 at 10:07













          Your Answer





          StackExchange.ifUsing("editor", function () {
          return StackExchange.using("mathjaxEditing", function () {
          StackExchange.MarkdownEditor.creationCallbacks.add(function (editor, postfix) {
          StackExchange.mathjaxEditing.prepareWmdForMathJax(editor, postfix, [["$", "$"], ["\\(","\\)"]]);
          });
          });
          }, "mathjax-editing");

          StackExchange.ready(function() {
          var channelOptions = {
          tags: "".split(" "),
          id: "69"
          };
          initTagRenderer("".split(" "), "".split(" "), channelOptions);

          StackExchange.using("externalEditor", function() {
          // Have to fire editor after snippets, if snippets enabled
          if (StackExchange.settings.snippets.snippetsEnabled) {
          StackExchange.using("snippets", function() {
          createEditor();
          });
          }
          else {
          createEditor();
          }
          });

          function createEditor() {
          StackExchange.prepareEditor({
          heartbeatType: 'answer',
          autoActivateHeartbeat: false,
          convertImagesToLinks: true,
          noModals: true,
          showLowRepImageUploadWarning: true,
          reputationToPostImages: 10,
          bindNavPrevention: true,
          postfix: "",
          imageUploader: {
          brandingHtml: "Powered by u003ca class="icon-imgur-white" href="https://imgur.com/"u003eu003c/au003e",
          contentPolicyHtml: "User contributions licensed under u003ca href="https://creativecommons.org/licenses/by-sa/3.0/"u003ecc by-sa 3.0 with attribution requiredu003c/au003e u003ca href="https://stackoverflow.com/legal/content-policy"u003e(content policy)u003c/au003e",
          allowUrls: true
          },
          noCode: true, onDemand: true,
          discardSelector: ".discard-answer"
          ,immediatelyShowMarkdownHelp:true
          });


          }
          });














          draft saved

          draft discarded


















          StackExchange.ready(
          function () {
          StackExchange.openid.initPostLogin('.new-post-login', 'https%3a%2f%2fmath.stackexchange.com%2fquestions%2f2214219%2fintegral-method-to-find-upper-and-lower-bounds-on-the-sum%23new-answer', 'question_page');
          }
          );

          Post as a guest















          Required, but never shown

























          1 Answer
          1






          active

          oldest

          votes








          1 Answer
          1






          active

          oldest

          votes









          active

          oldest

          votes






          active

          oldest

          votes









          0












          $begingroup$

          Hint. You may write
          $$
          begin{align}
          sum_{k=1}^nleft(3+frac1{k^3} right)&=left(3+frac1{1^3} right)+sum_{k=color{red}{2}}^nleft(3+frac1{k^3} right)
          \\&le left(3+frac1{1^3}right)+int_{color{red}{2-1}}^nleft(3+frac1{x^3} right)dx
          end{align}
          $$ then getting easily an upper bound.






          share|cite|improve this answer









          $endgroup$













          • $begingroup$
            thanks! Will I need to calculate the derivative of 5 in the integration part or just take the derivative of what's inside the summation and then multiply by 5? Also, for the lower bound, I can use the original summation without changing the bounds?
            $endgroup$
            – jenkins342
            Apr 2 '17 at 10:07


















          0












          $begingroup$

          Hint. You may write
          $$
          begin{align}
          sum_{k=1}^nleft(3+frac1{k^3} right)&=left(3+frac1{1^3} right)+sum_{k=color{red}{2}}^nleft(3+frac1{k^3} right)
          \\&le left(3+frac1{1^3}right)+int_{color{red}{2-1}}^nleft(3+frac1{x^3} right)dx
          end{align}
          $$ then getting easily an upper bound.






          share|cite|improve this answer









          $endgroup$













          • $begingroup$
            thanks! Will I need to calculate the derivative of 5 in the integration part or just take the derivative of what's inside the summation and then multiply by 5? Also, for the lower bound, I can use the original summation without changing the bounds?
            $endgroup$
            – jenkins342
            Apr 2 '17 at 10:07
















          0












          0








          0





          $begingroup$

          Hint. You may write
          $$
          begin{align}
          sum_{k=1}^nleft(3+frac1{k^3} right)&=left(3+frac1{1^3} right)+sum_{k=color{red}{2}}^nleft(3+frac1{k^3} right)
          \\&le left(3+frac1{1^3}right)+int_{color{red}{2-1}}^nleft(3+frac1{x^3} right)dx
          end{align}
          $$ then getting easily an upper bound.






          share|cite|improve this answer









          $endgroup$



          Hint. You may write
          $$
          begin{align}
          sum_{k=1}^nleft(3+frac1{k^3} right)&=left(3+frac1{1^3} right)+sum_{k=color{red}{2}}^nleft(3+frac1{k^3} right)
          \\&le left(3+frac1{1^3}right)+int_{color{red}{2-1}}^nleft(3+frac1{x^3} right)dx
          end{align}
          $$ then getting easily an upper bound.







          share|cite|improve this answer












          share|cite|improve this answer



          share|cite|improve this answer










          answered Apr 2 '17 at 9:45









          Olivier OloaOlivier Oloa

          109k17178294




          109k17178294












          • $begingroup$
            thanks! Will I need to calculate the derivative of 5 in the integration part or just take the derivative of what's inside the summation and then multiply by 5? Also, for the lower bound, I can use the original summation without changing the bounds?
            $endgroup$
            – jenkins342
            Apr 2 '17 at 10:07




















          • $begingroup$
            thanks! Will I need to calculate the derivative of 5 in the integration part or just take the derivative of what's inside the summation and then multiply by 5? Also, for the lower bound, I can use the original summation without changing the bounds?
            $endgroup$
            – jenkins342
            Apr 2 '17 at 10:07


















          $begingroup$
          thanks! Will I need to calculate the derivative of 5 in the integration part or just take the derivative of what's inside the summation and then multiply by 5? Also, for the lower bound, I can use the original summation without changing the bounds?
          $endgroup$
          – jenkins342
          Apr 2 '17 at 10:07






          $begingroup$
          thanks! Will I need to calculate the derivative of 5 in the integration part or just take the derivative of what's inside the summation and then multiply by 5? Also, for the lower bound, I can use the original summation without changing the bounds?
          $endgroup$
          – jenkins342
          Apr 2 '17 at 10:07




















          draft saved

          draft discarded




















































          Thanks for contributing an answer to Mathematics Stack Exchange!


          • Please be sure to answer the question. Provide details and share your research!

          But avoid



          • Asking for help, clarification, or responding to other answers.

          • Making statements based on opinion; back them up with references or personal experience.


          Use MathJax to format equations. MathJax reference.


          To learn more, see our tips on writing great answers.




          draft saved


          draft discarded














          StackExchange.ready(
          function () {
          StackExchange.openid.initPostLogin('.new-post-login', 'https%3a%2f%2fmath.stackexchange.com%2fquestions%2f2214219%2fintegral-method-to-find-upper-and-lower-bounds-on-the-sum%23new-answer', 'question_page');
          }
          );

          Post as a guest















          Required, but never shown





















































          Required, but never shown














          Required, but never shown












          Required, but never shown







          Required, but never shown

































          Required, but never shown














          Required, but never shown












          Required, but never shown







          Required, but never shown







          Popular posts from this blog

          Can a sorcerer learn a 5th-level spell early by creating spell slots using the Font of Magic feature?

          Does disintegrating a polymorphed enemy still kill it after the 2018 errata?

          A Topological Invariant for $pi_3(U(n))$